Allowed Publications
Slot System
Featured Buckets
Featured Buckets Admin

A Compulsion to Scratch, 
But Is There an Itch?

Article Type
Changed
Tue, 12/13/2016 - 12:08
Display Headline
A Compulsion to Scratch, 
But Is There an Itch?

ANSWER
The correct answer is neurotic excoriations (choice “d”), a chronic condition thought to be a psychologic process with dermatologic manifestations, consciously created by repetitive scratching and rubbing. Focally, it can manifest with skin alterations very similar to lichen simplex chronicus (choice “a”; also known as neurodermatitis), but the latter refers to very limited, localized processes and does not involve the psychiatric overlay seen with neurotic excoriations.

Patients with dermatitis artefacta (choice “b”; formerly called factitial dermatitis) consciously create their lesions for secondary gain, often using sharp objects such as nail files, kitchen utensils, or even shards of broken glass. Dermatitis artefacta lesions, which are relatively sparse and bizarre in appearance, can also be created by the application of caustic chemicals, or even by injection of foreign substances.

The differential rightly includes any number of skin conditions such as bullous pemphigoid (choice “c”). However, this was effectively ruled out by the biopsy and also by the morphology and extended chronicity of the patient’s complaint. 

DISCUSSION
Neurotic excoriations (NE) are usually created by unconscious picking, scratching, or rubbing. There may be a precipitating minor skin pathology (eg, insect bite, folliculitis or acne), but it can develop independent of any such process. Its origins can often be tied to upsetting life events, such as divorce, death, or early dementia.

More history taking from this patient and her family revealed that her skin problems began after her husband died in an accident, after which, according to her children, “she has never been the same.” Her picking accelerated when she moved to an assisted living setting.

Because patients create neurotic excoriations, their lesions have the quality of an “outside job,” with clean linear erosions, crusts, and scars that can be hypopigmented or hyperpigmented, depending on the patient’s skin type. Similar in size and shape, the lesions tend to be bilaterally and symmetrically distributed and confined to areas within easy reach, such as the extensor surfaces of the arms and the upper part of the back.

The vast majority of NE patients are adult women, though it is also seen in children as a manifestation of comorbid psychopathology or other psychosocial stressor.

TREATMENT
As one might expect, treatment of NE is difficult, particularly since many patients find it impossible to accept the role their mental state plays in the creation and perpetuation of their condition. In the best of all possible scenarios, the patient would be seen and followed by a psychiatrist, who would probably prescribe psychoactive medication. 

Failing that—or even in addition to that treatment—one could, at a minimum, find ways to distract the patient, trim her nails as much as possible, and/or place barriers between the offending nails and the skin in question.

Topical medications, such as steroid creams, are of very limited usefulness, as are oral antibiotics and antihistamines.           

Author and Disclosure Information

 

Joe R. Monroe, MPAS, PA

Issue
Clinician Reviews - 23(5)
Publications
Topics
Page Number
17
Legacy Keywords
dermatology, itch, lesions, scars, scabs, hyperkeratosis, biopsy, neurotic excoriations
Sections
Author and Disclosure Information

 

Joe R. Monroe, MPAS, PA

Author and Disclosure Information

 

Joe R. Monroe, MPAS, PA

ANSWER
The correct answer is neurotic excoriations (choice “d”), a chronic condition thought to be a psychologic process with dermatologic manifestations, consciously created by repetitive scratching and rubbing. Focally, it can manifest with skin alterations very similar to lichen simplex chronicus (choice “a”; also known as neurodermatitis), but the latter refers to very limited, localized processes and does not involve the psychiatric overlay seen with neurotic excoriations.

Patients with dermatitis artefacta (choice “b”; formerly called factitial dermatitis) consciously create their lesions for secondary gain, often using sharp objects such as nail files, kitchen utensils, or even shards of broken glass. Dermatitis artefacta lesions, which are relatively sparse and bizarre in appearance, can also be created by the application of caustic chemicals, or even by injection of foreign substances.

The differential rightly includes any number of skin conditions such as bullous pemphigoid (choice “c”). However, this was effectively ruled out by the biopsy and also by the morphology and extended chronicity of the patient’s complaint. 

DISCUSSION
Neurotic excoriations (NE) are usually created by unconscious picking, scratching, or rubbing. There may be a precipitating minor skin pathology (eg, insect bite, folliculitis or acne), but it can develop independent of any such process. Its origins can often be tied to upsetting life events, such as divorce, death, or early dementia.

More history taking from this patient and her family revealed that her skin problems began after her husband died in an accident, after which, according to her children, “she has never been the same.” Her picking accelerated when she moved to an assisted living setting.

Because patients create neurotic excoriations, their lesions have the quality of an “outside job,” with clean linear erosions, crusts, and scars that can be hypopigmented or hyperpigmented, depending on the patient’s skin type. Similar in size and shape, the lesions tend to be bilaterally and symmetrically distributed and confined to areas within easy reach, such as the extensor surfaces of the arms and the upper part of the back.

The vast majority of NE patients are adult women, though it is also seen in children as a manifestation of comorbid psychopathology or other psychosocial stressor.

TREATMENT
As one might expect, treatment of NE is difficult, particularly since many patients find it impossible to accept the role their mental state plays in the creation and perpetuation of their condition. In the best of all possible scenarios, the patient would be seen and followed by a psychiatrist, who would probably prescribe psychoactive medication. 

Failing that—or even in addition to that treatment—one could, at a minimum, find ways to distract the patient, trim her nails as much as possible, and/or place barriers between the offending nails and the skin in question.

Topical medications, such as steroid creams, are of very limited usefulness, as are oral antibiotics and antihistamines.           

ANSWER
The correct answer is neurotic excoriations (choice “d”), a chronic condition thought to be a psychologic process with dermatologic manifestations, consciously created by repetitive scratching and rubbing. Focally, it can manifest with skin alterations very similar to lichen simplex chronicus (choice “a”; also known as neurodermatitis), but the latter refers to very limited, localized processes and does not involve the psychiatric overlay seen with neurotic excoriations.

Patients with dermatitis artefacta (choice “b”; formerly called factitial dermatitis) consciously create their lesions for secondary gain, often using sharp objects such as nail files, kitchen utensils, or even shards of broken glass. Dermatitis artefacta lesions, which are relatively sparse and bizarre in appearance, can also be created by the application of caustic chemicals, or even by injection of foreign substances.

The differential rightly includes any number of skin conditions such as bullous pemphigoid (choice “c”). However, this was effectively ruled out by the biopsy and also by the morphology and extended chronicity of the patient’s complaint. 

DISCUSSION
Neurotic excoriations (NE) are usually created by unconscious picking, scratching, or rubbing. There may be a precipitating minor skin pathology (eg, insect bite, folliculitis or acne), but it can develop independent of any such process. Its origins can often be tied to upsetting life events, such as divorce, death, or early dementia.

More history taking from this patient and her family revealed that her skin problems began after her husband died in an accident, after which, according to her children, “she has never been the same.” Her picking accelerated when she moved to an assisted living setting.

Because patients create neurotic excoriations, their lesions have the quality of an “outside job,” with clean linear erosions, crusts, and scars that can be hypopigmented or hyperpigmented, depending on the patient’s skin type. Similar in size and shape, the lesions tend to be bilaterally and symmetrically distributed and confined to areas within easy reach, such as the extensor surfaces of the arms and the upper part of the back.

The vast majority of NE patients are adult women, though it is also seen in children as a manifestation of comorbid psychopathology or other psychosocial stressor.

TREATMENT
As one might expect, treatment of NE is difficult, particularly since many patients find it impossible to accept the role their mental state plays in the creation and perpetuation of their condition. In the best of all possible scenarios, the patient would be seen and followed by a psychiatrist, who would probably prescribe psychoactive medication. 

Failing that—or even in addition to that treatment—one could, at a minimum, find ways to distract the patient, trim her nails as much as possible, and/or place barriers between the offending nails and the skin in question.

Topical medications, such as steroid creams, are of very limited usefulness, as are oral antibiotics and antihistamines.           

Issue
Clinician Reviews - 23(5)
Issue
Clinician Reviews - 23(5)
Page Number
17
Page Number
17
Publications
Publications
Topics
Article Type
Display Headline
A Compulsion to Scratch, 
But Is There an Itch?
Display Headline
A Compulsion to Scratch, 
But Is There an Itch?
Legacy Keywords
dermatology, itch, lesions, scars, scabs, hyperkeratosis, biopsy, neurotic excoriations
Legacy Keywords
dermatology, itch, lesions, scars, scabs, hyperkeratosis, biopsy, neurotic excoriations
Sections
Questionnaire Body

 

 

At her daughters’ insistence, this 69-year-old woman requests referral to dermatology for a skin condition that has been present for at least 20 years. During that time, she has seen many medical providers (including dermatologists) and has tried many different treatments (eg, creams, oral antibiotics, oral steroids, and antihistamines). While some of these helped a bit, most did not help at all—nor did the constant nagging at the patient by family and caregivers. Nonetheless, her daughters feel strongly that their mother is perpetuating the problem with her “scratching and picking.” They have observed that when she is able to leave her arms alone, the improvement in her skin is dramatic. For example, years ago, she broke her wrist and was placed in a cast for six weeks; when it was removed, the affected arm was completely clear (except for multiple old scars). Everyone, including the patient, was ecstatic—but a week later, the lesions returned. The extensor aspects of both arms and hands are covered with linear excoriations, scars, and scabs, with focal hyperpigmentation in many of the excoriated areas. Overall, the skin in these areas is re-markably thickened and focally shiny. Her skin elsewhere—such as her palms and the volar aspects of her arms—is relatively clear. Throughout the examination, the patient’s hands never stop rubbing and scratching her arms, even as she weakly denies doing so. “Whatever happens, I’m not going to see a shrink,” she says. Clearly, a biopsy is in order, with a sample taken from a typical section of her forearm. The results show minimal changes but demonstrate hyperkeratosis. Blood work, including a complete metabolic profile and complete blood count, fail to show any evidence of systemic disease.

 

Disallow All Ads

An Encounter With Unflattering Light

Article Type
Changed
Tue, 12/13/2016 - 12:08
Display Headline
An Encounter With Unflattering Light

ANSWER
The correct answer is dermatoheliosis (choice “d”), also correctly termed photoaging. This condition manifests as a number of specific skin changes, including the items named (choices “a,” “b,” and “c”)—all of which were present on this patient.

DISCUSSION
The consequences of chronic overexposure to UV radiation constitute the most common reason patients present to dermatology practices in the United States. The bulk of this damage takes decades to appear, by which time patients have forgotten about their earlier sun exposure (in fact, they often deny any exposure) and even the painful sunburns that taught them to avoid the sun in the first place.

In general, the effects of sunburns sustained in childhood or young adulthood do not usually manifest until the patient is in his/her 50s or 60s, although patients who are less sun-tolerant (our definition of “fair”) may show signs of damage considerably earlier.

However, with the popularity of artificial tanning among teenagers (and even preteens in some cases), evidence of sun damage is being seen at younger ages than ever. Basal cell carcinomas, once unheard of in teenagers, are being found with increasing frequency in this age-group. In patients ages 12 to 15, there has been a 100-fold increase in the incidence of melanoma—theorized to be due, in part, to the effects of artificial tanning.

This particular patient is typical of cases in which sun damage was obtained more passively. At one time in the US, having a tan was decidedly unfashionable; it marked one as a member of “the lower classes.” But that all began to change after WWI: Hemlines and hairlines rose, Prohibition created a new generation of drinkers and scofflaws, clothing began to be more revealing, and suddenly it was fashionable for women to shave their legs and get a tan.

About that same time, many men began to ignore the long-held tradition of wearing hats and long sleeves when outside, inevitably tanned, and thus gained approval from the opposite sex. Most went off to war in the 1940s, many to the Pacific theater, where they had even more exposure to the sun.

Following WWII, a great number of these men returned to their jobs as farmers, ranchers, or construction workers. Golfing became the “in” sport during leisure time. It’s this generation we’re seeing now for sun-related pathology. Even if they had been inclined to use it, effective sunscreen was not generally available until the early 1970s.

The patient depicted here has a typical collection of the pre-cancerous sun damage known as dermatoheliosis: solar elastosis, actinic keratoses, telangiectasias, and solar atrophy (which affects the arms more than the face). The latter, along with the effects of wind, heat, cold, smoking, and drinking alcohol, constitute the main causes of extrinsic aging.

TREATMENT
Short of heroic efforts, not much will be done for this patient’s dermatoheliosis. However, he was strongly advised to return to dermatology twice a year to watch for the arrival of the basal cell and squamous cell carcinomas that are almost certainly headed his way.

 

Author and Disclosure Information

 

Joe R. Monroe, MPAS, PA

Issue
Clinician Reviews - 23(7)
Publications
Topics
Page Number
17
Legacy Keywords
dermatology, dermatoheliosis, photoaging, chicken skin, telangiectasia, UV radiation, exposure, ultraviolet light, solar elastosis, actinic keratoses, telangiectasias, solar atrophy
Sections
Author and Disclosure Information

 

Joe R. Monroe, MPAS, PA

Author and Disclosure Information

 

Joe R. Monroe, MPAS, PA

ANSWER
The correct answer is dermatoheliosis (choice “d”), also correctly termed photoaging. This condition manifests as a number of specific skin changes, including the items named (choices “a,” “b,” and “c”)—all of which were present on this patient.

DISCUSSION
The consequences of chronic overexposure to UV radiation constitute the most common reason patients present to dermatology practices in the United States. The bulk of this damage takes decades to appear, by which time patients have forgotten about their earlier sun exposure (in fact, they often deny any exposure) and even the painful sunburns that taught them to avoid the sun in the first place.

In general, the effects of sunburns sustained in childhood or young adulthood do not usually manifest until the patient is in his/her 50s or 60s, although patients who are less sun-tolerant (our definition of “fair”) may show signs of damage considerably earlier.

However, with the popularity of artificial tanning among teenagers (and even preteens in some cases), evidence of sun damage is being seen at younger ages than ever. Basal cell carcinomas, once unheard of in teenagers, are being found with increasing frequency in this age-group. In patients ages 12 to 15, there has been a 100-fold increase in the incidence of melanoma—theorized to be due, in part, to the effects of artificial tanning.

This particular patient is typical of cases in which sun damage was obtained more passively. At one time in the US, having a tan was decidedly unfashionable; it marked one as a member of “the lower classes.” But that all began to change after WWI: Hemlines and hairlines rose, Prohibition created a new generation of drinkers and scofflaws, clothing began to be more revealing, and suddenly it was fashionable for women to shave their legs and get a tan.

About that same time, many men began to ignore the long-held tradition of wearing hats and long sleeves when outside, inevitably tanned, and thus gained approval from the opposite sex. Most went off to war in the 1940s, many to the Pacific theater, where they had even more exposure to the sun.

Following WWII, a great number of these men returned to their jobs as farmers, ranchers, or construction workers. Golfing became the “in” sport during leisure time. It’s this generation we’re seeing now for sun-related pathology. Even if they had been inclined to use it, effective sunscreen was not generally available until the early 1970s.

The patient depicted here has a typical collection of the pre-cancerous sun damage known as dermatoheliosis: solar elastosis, actinic keratoses, telangiectasias, and solar atrophy (which affects the arms more than the face). The latter, along with the effects of wind, heat, cold, smoking, and drinking alcohol, constitute the main causes of extrinsic aging.

TREATMENT
Short of heroic efforts, not much will be done for this patient’s dermatoheliosis. However, he was strongly advised to return to dermatology twice a year to watch for the arrival of the basal cell and squamous cell carcinomas that are almost certainly headed his way.

 

ANSWER
The correct answer is dermatoheliosis (choice “d”), also correctly termed photoaging. This condition manifests as a number of specific skin changes, including the items named (choices “a,” “b,” and “c”)—all of which were present on this patient.

DISCUSSION
The consequences of chronic overexposure to UV radiation constitute the most common reason patients present to dermatology practices in the United States. The bulk of this damage takes decades to appear, by which time patients have forgotten about their earlier sun exposure (in fact, they often deny any exposure) and even the painful sunburns that taught them to avoid the sun in the first place.

In general, the effects of sunburns sustained in childhood or young adulthood do not usually manifest until the patient is in his/her 50s or 60s, although patients who are less sun-tolerant (our definition of “fair”) may show signs of damage considerably earlier.

However, with the popularity of artificial tanning among teenagers (and even preteens in some cases), evidence of sun damage is being seen at younger ages than ever. Basal cell carcinomas, once unheard of in teenagers, are being found with increasing frequency in this age-group. In patients ages 12 to 15, there has been a 100-fold increase in the incidence of melanoma—theorized to be due, in part, to the effects of artificial tanning.

This particular patient is typical of cases in which sun damage was obtained more passively. At one time in the US, having a tan was decidedly unfashionable; it marked one as a member of “the lower classes.” But that all began to change after WWI: Hemlines and hairlines rose, Prohibition created a new generation of drinkers and scofflaws, clothing began to be more revealing, and suddenly it was fashionable for women to shave their legs and get a tan.

About that same time, many men began to ignore the long-held tradition of wearing hats and long sleeves when outside, inevitably tanned, and thus gained approval from the opposite sex. Most went off to war in the 1940s, many to the Pacific theater, where they had even more exposure to the sun.

Following WWII, a great number of these men returned to their jobs as farmers, ranchers, or construction workers. Golfing became the “in” sport during leisure time. It’s this generation we’re seeing now for sun-related pathology. Even if they had been inclined to use it, effective sunscreen was not generally available until the early 1970s.

The patient depicted here has a typical collection of the pre-cancerous sun damage known as dermatoheliosis: solar elastosis, actinic keratoses, telangiectasias, and solar atrophy (which affects the arms more than the face). The latter, along with the effects of wind, heat, cold, smoking, and drinking alcohol, constitute the main causes of extrinsic aging.

TREATMENT
Short of heroic efforts, not much will be done for this patient’s dermatoheliosis. However, he was strongly advised to return to dermatology twice a year to watch for the arrival of the basal cell and squamous cell carcinomas that are almost certainly headed his way.

 

Issue
Clinician Reviews - 23(7)
Issue
Clinician Reviews - 23(7)
Page Number
17
Page Number
17
Publications
Publications
Topics
Article Type
Display Headline
An Encounter With Unflattering Light
Display Headline
An Encounter With Unflattering Light
Legacy Keywords
dermatology, dermatoheliosis, photoaging, chicken skin, telangiectasia, UV radiation, exposure, ultraviolet light, solar elastosis, actinic keratoses, telangiectasias, solar atrophy
Legacy Keywords
dermatology, dermatoheliosis, photoaging, chicken skin, telangiectasia, UV radiation, exposure, ultraviolet light, solar elastosis, actinic keratoses, telangiectasias, solar atrophy
Sections
Questionnaire Body

 

 

During a recent trip, this 77-year-old man stayed in a hotel in which the bathroom lighting was considerably brighter than that at home—allowing him to see a number of skin changes he hadn’t noticed before. As a result, he presents to dermatology for an evaluation. The patient’s forehead, as well as his cheeks and nose, look curiously mottled (pink and white), with a rough, scar-like, pebbly surface that resembles chicken skin. There are also numerous 1- to 3-mm rough, scaly, papular lesions and multiple faint telangiectasias. In sun-exposed areas, such as his hands and arms, the skin is rough, dry, and exceptionally thin, with light and dark color changes; this is in sharp contrast to the relatively pristine texture and uniformly light color of the volar forearms and other areas that are not exposed to the sun. History taking reveals that, as a young man, the patient spent a great deal of time outdoors, both at work and in his free time. He never wore a hat or used any other form of sun protection. Since age 50, he has had several skin cancers removed from his face and back. Despite this, he is not seeing a dermatology provider regularly.

 

Disallow All Ads

Wife Wants Husband’s “Zits” Gone!

Article Type
Changed
Tue, 12/13/2016 - 12:08
Display Headline
Wife Wants Husband’s “Zits” Gone!

ANSWER
The correct answer is dilated pore of Winer (choice “b”), a hair structure anomaly discussed below. Sebaceous cysts (choice “a”) often present with a surface punctum, but the depth and appearance of this pore are not consistent with a simple punctum. The same could be said of the other two choices: ice-pick scar secondary to acne (choice “c”) and ingrown hair (choice “d”).

DISCUSSION
Dilated pore of Winer is actually a tumor of the intraepidermal follicle and related infundibulum of the pilosebaceous apparatus, a fact confirmed by immunohistochemical studies. It has no implication for health, but its appearance is occasionally distressing. Unfortunately, this patient has matching dilated pores on either side of his nose.

These scar-like pits are most commonly seen on the face, especially the maxillae. Even though they resemble one another, dilated pore of Winer differs significantly from a simple comedone: The former is considerably deeper, as well as markedly different in structure.

TREATMENT
The only effective treatment for dilated pore of Winer is surgical excision, which is easily accomplished under local anesthesia. A 4- to 5-mm punch biopsy tool is introduced into the skin at the same angle as the course of the pore, then taken down to adipose tissue, which ensures complete removal. Two interrupted skin sutures serve to convert the round punch defect into a linear wound, preferably matching skin tension lines. The tissue thus removed is always sent for pathologic examination to rule out basal cell carcinoma.

But for the vast majority of patients affected by dilated pore of Winer, the best treatment is to leave the lesions alone.

Author and Disclosure Information

 

Joe R. Monroe, MPAS, PA

Issue
Clinician Reviews - 23(6)
Publications
Topics
Page Number
14
Legacy Keywords
dermatology, pimple, zits, acne, dilated pore of Winer, maxillae,
Sections
Author and Disclosure Information

 

Joe R. Monroe, MPAS, PA

Author and Disclosure Information

 

Joe R. Monroe, MPAS, PA

ANSWER
The correct answer is dilated pore of Winer (choice “b”), a hair structure anomaly discussed below. Sebaceous cysts (choice “a”) often present with a surface punctum, but the depth and appearance of this pore are not consistent with a simple punctum. The same could be said of the other two choices: ice-pick scar secondary to acne (choice “c”) and ingrown hair (choice “d”).

DISCUSSION
Dilated pore of Winer is actually a tumor of the intraepidermal follicle and related infundibulum of the pilosebaceous apparatus, a fact confirmed by immunohistochemical studies. It has no implication for health, but its appearance is occasionally distressing. Unfortunately, this patient has matching dilated pores on either side of his nose.

These scar-like pits are most commonly seen on the face, especially the maxillae. Even though they resemble one another, dilated pore of Winer differs significantly from a simple comedone: The former is considerably deeper, as well as markedly different in structure.

TREATMENT
The only effective treatment for dilated pore of Winer is surgical excision, which is easily accomplished under local anesthesia. A 4- to 5-mm punch biopsy tool is introduced into the skin at the same angle as the course of the pore, then taken down to adipose tissue, which ensures complete removal. Two interrupted skin sutures serve to convert the round punch defect into a linear wound, preferably matching skin tension lines. The tissue thus removed is always sent for pathologic examination to rule out basal cell carcinoma.

But for the vast majority of patients affected by dilated pore of Winer, the best treatment is to leave the lesions alone.

ANSWER
The correct answer is dilated pore of Winer (choice “b”), a hair structure anomaly discussed below. Sebaceous cysts (choice “a”) often present with a surface punctum, but the depth and appearance of this pore are not consistent with a simple punctum. The same could be said of the other two choices: ice-pick scar secondary to acne (choice “c”) and ingrown hair (choice “d”).

DISCUSSION
Dilated pore of Winer is actually a tumor of the intraepidermal follicle and related infundibulum of the pilosebaceous apparatus, a fact confirmed by immunohistochemical studies. It has no implication for health, but its appearance is occasionally distressing. Unfortunately, this patient has matching dilated pores on either side of his nose.

These scar-like pits are most commonly seen on the face, especially the maxillae. Even though they resemble one another, dilated pore of Winer differs significantly from a simple comedone: The former is considerably deeper, as well as markedly different in structure.

TREATMENT
The only effective treatment for dilated pore of Winer is surgical excision, which is easily accomplished under local anesthesia. A 4- to 5-mm punch biopsy tool is introduced into the skin at the same angle as the course of the pore, then taken down to adipose tissue, which ensures complete removal. Two interrupted skin sutures serve to convert the round punch defect into a linear wound, preferably matching skin tension lines. The tissue thus removed is always sent for pathologic examination to rule out basal cell carcinoma.

But for the vast majority of patients affected by dilated pore of Winer, the best treatment is to leave the lesions alone.

Issue
Clinician Reviews - 23(6)
Issue
Clinician Reviews - 23(6)
Page Number
14
Page Number
14
Publications
Publications
Topics
Article Type
Display Headline
Wife Wants Husband’s “Zits” Gone!
Display Headline
Wife Wants Husband’s “Zits” Gone!
Legacy Keywords
dermatology, pimple, zits, acne, dilated pore of Winer, maxillae,
Legacy Keywords
dermatology, pimple, zits, acne, dilated pore of Winer, maxillae,
Sections
Questionnaire Body

 

 

A 52-year-old man self-refers to dermatology, at his wife’s insistence, for evaluation of “big black-heads” that have been present on his maxillae for as long as he can remember. Periodically, he ex-presses cheesy, odoriferous material from them. He denies ever experiencing trauma in the area, and there is no history of other skin problems (eg, acne). His wife wants him to get these “black-heads” removed, because there is “dirt” in them. Small “holes” are seen on each side of the nose, about 3 cm lateral to midline. Each lesion is 2 to 3 mm wide and obviously deep. There is no comedonal material or protruding hair seen in the lesions; the surrounding skin is unchanged. Induration is absent in or around the lesions. No signs of active acne are seen elsewhere.

 

Disallow All Ads

After 15 Years, Still Losing Hair, Only Faster

Article Type
Changed
Tue, 12/13/2016 - 12:08
Display Headline
After 15 Years, Still Losing Hair, Only Faster

ANSWER
This is a classic clinical picture of androgenetic alopecia (choice “a”). See discussion for more details.

Alopecia areata (choice “b”) usually manifests acutely and leads to complete hair loss in a well-defined, annular pattern. It typically resolves on its own, with or without treatment.

Telogen effluvium (choice “c”) involves generalized hair loss without a pattern. The hair is actually “lost,” meaning markedly increased amounts of hair are seen in the comb, brush, sink, or shower. This results in an increasingly visible scalp.

Without a clear clinical picture of alopecia, a biopsy might have been indicated—primarily to rule out conditions such as lupus erythematosus (choice “d”), which can involve hair loss of various kinds. The negative ANA result obtained by the patient’s primary care provider helped rule out this diagnosis.

DISCUSSION
Androgenetic alopecia (AGA) affects both men and women, though the latter begin to develop it about 10 years later, on average, than men do. Among women, 13% develop AGA before menopause, while 75% note its appearance postmenopausally.

In both sexes, AGA results from the gradual conversion of terminal hairs to vellus hairs, with miniaturization of the follicles. Hair loss in men starts in the vertex, followed by bitemporal recession. In women, AGA primarily affects the crown of the scalp, often with partial preservation of the frontal hairline.

Dihydrotestosterone (DHT) appears to be the main culprit; testosterone is converted to DHT by means of the enzyme 5α-reductase. One of the most effective medications for AGA in men has been finasteride, which blocks the effects of 5α-reductase and can at least slow the rate of hair loss. Unfortunately, finasteride does not appear to be effective in treating AGA in women.

Women do, however, appear to respond to minoxidil, a topically applied solution, better than men. The response is moderate at best, and any hair gained is lost if the treatment is discontinued. Interestingly, the stronger 5% solution of minoxidil in women does not produce any demonstrable improvement over that seen with the 2% solution.

From a practical diagnostic standpoint, it is quite common for women with longstanding mild to moderate AGA to present with an acute episode of telogen effluvium (TE), in which hair all over the scalp falls out. Careful history taking is necessary to tease these stories apart, since TE will typically resolve on its own. The most common causes of TE, in my experience, are stress, extreme weight loss, and as a consequence of general anesthesia. For unknown reasons, TE is almost nonexistent in men.

TREATMENT
This patient chose to use 5% OTC minoxidil, an antihypertensive with an unknown mode of action in AGA. She’ll confine its application to the affected areas of the scalp, since unwanted hair growth has been reported on the face with the use of this medication.

Author and Disclosure Information

 

 

Issue
Clinician Reviews - 23(4)
Publications
Topics
Page Number
13
Legacy Keywords
hair loss, dermatology, menopause, scalp, thinning, Androgenetic alopecia, Alopecia areata, Telogen effluvium, Lupus erythematosus, Dihydrotestosterone
Sections
Author and Disclosure Information

 

 

Author and Disclosure Information

 

 

ANSWER
This is a classic clinical picture of androgenetic alopecia (choice “a”). See discussion for more details.

Alopecia areata (choice “b”) usually manifests acutely and leads to complete hair loss in a well-defined, annular pattern. It typically resolves on its own, with or without treatment.

Telogen effluvium (choice “c”) involves generalized hair loss without a pattern. The hair is actually “lost,” meaning markedly increased amounts of hair are seen in the comb, brush, sink, or shower. This results in an increasingly visible scalp.

Without a clear clinical picture of alopecia, a biopsy might have been indicated—primarily to rule out conditions such as lupus erythematosus (choice “d”), which can involve hair loss of various kinds. The negative ANA result obtained by the patient’s primary care provider helped rule out this diagnosis.

DISCUSSION
Androgenetic alopecia (AGA) affects both men and women, though the latter begin to develop it about 10 years later, on average, than men do. Among women, 13% develop AGA before menopause, while 75% note its appearance postmenopausally.

In both sexes, AGA results from the gradual conversion of terminal hairs to vellus hairs, with miniaturization of the follicles. Hair loss in men starts in the vertex, followed by bitemporal recession. In women, AGA primarily affects the crown of the scalp, often with partial preservation of the frontal hairline.

Dihydrotestosterone (DHT) appears to be the main culprit; testosterone is converted to DHT by means of the enzyme 5α-reductase. One of the most effective medications for AGA in men has been finasteride, which blocks the effects of 5α-reductase and can at least slow the rate of hair loss. Unfortunately, finasteride does not appear to be effective in treating AGA in women.

Women do, however, appear to respond to minoxidil, a topically applied solution, better than men. The response is moderate at best, and any hair gained is lost if the treatment is discontinued. Interestingly, the stronger 5% solution of minoxidil in women does not produce any demonstrable improvement over that seen with the 2% solution.

From a practical diagnostic standpoint, it is quite common for women with longstanding mild to moderate AGA to present with an acute episode of telogen effluvium (TE), in which hair all over the scalp falls out. Careful history taking is necessary to tease these stories apart, since TE will typically resolve on its own. The most common causes of TE, in my experience, are stress, extreme weight loss, and as a consequence of general anesthesia. For unknown reasons, TE is almost nonexistent in men.

TREATMENT
This patient chose to use 5% OTC minoxidil, an antihypertensive with an unknown mode of action in AGA. She’ll confine its application to the affected areas of the scalp, since unwanted hair growth has been reported on the face with the use of this medication.

ANSWER
This is a classic clinical picture of androgenetic alopecia (choice “a”). See discussion for more details.

Alopecia areata (choice “b”) usually manifests acutely and leads to complete hair loss in a well-defined, annular pattern. It typically resolves on its own, with or without treatment.

Telogen effluvium (choice “c”) involves generalized hair loss without a pattern. The hair is actually “lost,” meaning markedly increased amounts of hair are seen in the comb, brush, sink, or shower. This results in an increasingly visible scalp.

Without a clear clinical picture of alopecia, a biopsy might have been indicated—primarily to rule out conditions such as lupus erythematosus (choice “d”), which can involve hair loss of various kinds. The negative ANA result obtained by the patient’s primary care provider helped rule out this diagnosis.

DISCUSSION
Androgenetic alopecia (AGA) affects both men and women, though the latter begin to develop it about 10 years later, on average, than men do. Among women, 13% develop AGA before menopause, while 75% note its appearance postmenopausally.

In both sexes, AGA results from the gradual conversion of terminal hairs to vellus hairs, with miniaturization of the follicles. Hair loss in men starts in the vertex, followed by bitemporal recession. In women, AGA primarily affects the crown of the scalp, often with partial preservation of the frontal hairline.

Dihydrotestosterone (DHT) appears to be the main culprit; testosterone is converted to DHT by means of the enzyme 5α-reductase. One of the most effective medications for AGA in men has been finasteride, which blocks the effects of 5α-reductase and can at least slow the rate of hair loss. Unfortunately, finasteride does not appear to be effective in treating AGA in women.

Women do, however, appear to respond to minoxidil, a topically applied solution, better than men. The response is moderate at best, and any hair gained is lost if the treatment is discontinued. Interestingly, the stronger 5% solution of minoxidil in women does not produce any demonstrable improvement over that seen with the 2% solution.

From a practical diagnostic standpoint, it is quite common for women with longstanding mild to moderate AGA to present with an acute episode of telogen effluvium (TE), in which hair all over the scalp falls out. Careful history taking is necessary to tease these stories apart, since TE will typically resolve on its own. The most common causes of TE, in my experience, are stress, extreme weight loss, and as a consequence of general anesthesia. For unknown reasons, TE is almost nonexistent in men.

TREATMENT
This patient chose to use 5% OTC minoxidil, an antihypertensive with an unknown mode of action in AGA. She’ll confine its application to the affected areas of the scalp, since unwanted hair growth has been reported on the face with the use of this medication.

Issue
Clinician Reviews - 23(4)
Issue
Clinician Reviews - 23(4)
Page Number
13
Page Number
13
Publications
Publications
Topics
Article Type
Display Headline
After 15 Years, Still Losing Hair, Only Faster
Display Headline
After 15 Years, Still Losing Hair, Only Faster
Legacy Keywords
hair loss, dermatology, menopause, scalp, thinning, Androgenetic alopecia, Alopecia areata, Telogen effluvium, Lupus erythematosus, Dihydrotestosterone
Legacy Keywords
hair loss, dermatology, menopause, scalp, thinning, Androgenetic alopecia, Alopecia areata, Telogen effluvium, Lupus erythematosus, Dihydrotestosterone
Sections
Questionnaire Body

 

 

A 43-year-old woman presents to dermatology with the extremely common complaint of hair loss. The problem is not new; she first noticed it 15 years ago. But the loss has now progressed to such an extent that the patient consulted her primary care provider. Blood tests were ordered, including complete blood count, antinuclear antibody (ANA), and thy-roid-stimulating hormone; all results were within normal limits. And so she decided to seek a specialist’s assessment. The patient is going through menopause—without the aid of medication—and claims to be otherwise healthy. She denies finding increased amounts of lost hair in her comb, brush, shower, or sink. She further denies any symptoms in her scalp. Her mother and one sister had similar problems with their scalp hair. Examination reveals extensive thinning of hair, which is almost totally confined to the crown of her scalp, with faint but obvious preservation of a thin band of the frontal hairline. There is no appreciable disruption of the skin surface in the scalp (eg, scaling, redness, edema, or scarring).

 

Disallow All Ads

Thumb: Scaling with Pitted Nail Plate

Article Type
Changed
Tue, 12/13/2016 - 12:08
Display Headline
Thumb: Scaling with Pitted Nail Plate

ANSWER
The most likely diagnosis is psoriasis (choice “d”), which can manifest with localized involvement (see discussion below).

Fungal infection (choice “a”) is unlikely, given the negative results of the KOH prep and total lack of response to treatment for that diagnosis.

Eczema (choice “b”) does not manifest as such thick, adherent scale. If any nail changes were involved, they would likely consist of transverse nail ridges.

A variant of squamous cell carcinoma (SCC; choice “c”)—caused by human papillomavirus (HPV), for example—can produce somewhat similar changes in the skin. But it would be unlikely to lead to nail changes, and it would not be intermittent, as this rash was in apparent response to OTC cream.

DISCUSSION
A punch biopsy is sometimes required to confirm the diagnosis of psoriasis, but this combination of skin and nail changes is quite suggestive of that entity. In this case, the confirmation was made by a different route: The rheumatologist judged that the patient’s arthritis was psoriatic in nature. The arthritis was treated with methotrexate, which soon cleared the skin disease without any help from dermatology.

This case serves to reinforce the possible mind-body role of stress in the genesis of this disease; however, at least 30% of the time, there is a positive family history. It also demonstrates the seemingly contradictory fact that the severity of the psoriasis doesn’t always correlate with the presence or absence of psoriatic arthritis.

Adding to the difficulty of making the diagnosis of psoriasis is the localized distribution of the scales, since it can usually be corroborated by finding lesions in their usual haunts—such as the extensor surfaces of arms, legs, on the trunk, or in the scalp. Cases such as this one force the provider to carefully consider the differential, which includes fungal infection. But the dermatophytes that cause ordinary fungal infections have to come from predictable sources (eg, children, pets, or farm animals), all missing from this patient’s history.

Had the rash been “fixed” (unchanging), the diagnosis of SCC would have to be considered more carefully. Superficial SCC is called Bowen’s disease, which, in cases such as this, is usually caused by HPV, not by the more typical overexposure to UV light. Though superficial, Bowen’s disease can become focally invasive and can even metastasize.

Had this patient not been treated with methotrexate, we would probably have used topical class 1 corticosteroid creams, which would have had a good chance to improve his skin but not his nail. The patient was also counseled regarding the role of stress and increased alcohol intake in the worsening of his disease. His prognosis for skin and joint disease is decent, though he will probably experience recurrences.

Author and Disclosure Information

 

Joe R. Monroe, MPAS, PA

Issue
Clinician Reviews - 23(3)
Publications
Topics
Page Number
11
Legacy Keywords
dermatology, scaling, thumb, hydrocortisone, cream, fungal infection, clotrimazole/betamethasone cream, terbinafine, stress, arthritis, scales, KOH prep, discoloration, Fungal infection, Eczema, Squamous cell carcinoma, Psoriasis
Sections
Author and Disclosure Information

 

Joe R. Monroe, MPAS, PA

Author and Disclosure Information

 

Joe R. Monroe, MPAS, PA

ANSWER
The most likely diagnosis is psoriasis (choice “d”), which can manifest with localized involvement (see discussion below).

Fungal infection (choice “a”) is unlikely, given the negative results of the KOH prep and total lack of response to treatment for that diagnosis.

Eczema (choice “b”) does not manifest as such thick, adherent scale. If any nail changes were involved, they would likely consist of transverse nail ridges.

A variant of squamous cell carcinoma (SCC; choice “c”)—caused by human papillomavirus (HPV), for example—can produce somewhat similar changes in the skin. But it would be unlikely to lead to nail changes, and it would not be intermittent, as this rash was in apparent response to OTC cream.

DISCUSSION
A punch biopsy is sometimes required to confirm the diagnosis of psoriasis, but this combination of skin and nail changes is quite suggestive of that entity. In this case, the confirmation was made by a different route: The rheumatologist judged that the patient’s arthritis was psoriatic in nature. The arthritis was treated with methotrexate, which soon cleared the skin disease without any help from dermatology.

This case serves to reinforce the possible mind-body role of stress in the genesis of this disease; however, at least 30% of the time, there is a positive family history. It also demonstrates the seemingly contradictory fact that the severity of the psoriasis doesn’t always correlate with the presence or absence of psoriatic arthritis.

Adding to the difficulty of making the diagnosis of psoriasis is the localized distribution of the scales, since it can usually be corroborated by finding lesions in their usual haunts—such as the extensor surfaces of arms, legs, on the trunk, or in the scalp. Cases such as this one force the provider to carefully consider the differential, which includes fungal infection. But the dermatophytes that cause ordinary fungal infections have to come from predictable sources (eg, children, pets, or farm animals), all missing from this patient’s history.

Had the rash been “fixed” (unchanging), the diagnosis of SCC would have to be considered more carefully. Superficial SCC is called Bowen’s disease, which, in cases such as this, is usually caused by HPV, not by the more typical overexposure to UV light. Though superficial, Bowen’s disease can become focally invasive and can even metastasize.

Had this patient not been treated with methotrexate, we would probably have used topical class 1 corticosteroid creams, which would have had a good chance to improve his skin but not his nail. The patient was also counseled regarding the role of stress and increased alcohol intake in the worsening of his disease. His prognosis for skin and joint disease is decent, though he will probably experience recurrences.

ANSWER
The most likely diagnosis is psoriasis (choice “d”), which can manifest with localized involvement (see discussion below).

Fungal infection (choice “a”) is unlikely, given the negative results of the KOH prep and total lack of response to treatment for that diagnosis.

Eczema (choice “b”) does not manifest as such thick, adherent scale. If any nail changes were involved, they would likely consist of transverse nail ridges.

A variant of squamous cell carcinoma (SCC; choice “c”)—caused by human papillomavirus (HPV), for example—can produce somewhat similar changes in the skin. But it would be unlikely to lead to nail changes, and it would not be intermittent, as this rash was in apparent response to OTC cream.

DISCUSSION
A punch biopsy is sometimes required to confirm the diagnosis of psoriasis, but this combination of skin and nail changes is quite suggestive of that entity. In this case, the confirmation was made by a different route: The rheumatologist judged that the patient’s arthritis was psoriatic in nature. The arthritis was treated with methotrexate, which soon cleared the skin disease without any help from dermatology.

This case serves to reinforce the possible mind-body role of stress in the genesis of this disease; however, at least 30% of the time, there is a positive family history. It also demonstrates the seemingly contradictory fact that the severity of the psoriasis doesn’t always correlate with the presence or absence of psoriatic arthritis.

Adding to the difficulty of making the diagnosis of psoriasis is the localized distribution of the scales, since it can usually be corroborated by finding lesions in their usual haunts—such as the extensor surfaces of arms, legs, on the trunk, or in the scalp. Cases such as this one force the provider to carefully consider the differential, which includes fungal infection. But the dermatophytes that cause ordinary fungal infections have to come from predictable sources (eg, children, pets, or farm animals), all missing from this patient’s history.

Had the rash been “fixed” (unchanging), the diagnosis of SCC would have to be considered more carefully. Superficial SCC is called Bowen’s disease, which, in cases such as this, is usually caused by HPV, not by the more typical overexposure to UV light. Though superficial, Bowen’s disease can become focally invasive and can even metastasize.

Had this patient not been treated with methotrexate, we would probably have used topical class 1 corticosteroid creams, which would have had a good chance to improve his skin but not his nail. The patient was also counseled regarding the role of stress and increased alcohol intake in the worsening of his disease. His prognosis for skin and joint disease is decent, though he will probably experience recurrences.

Issue
Clinician Reviews - 23(3)
Issue
Clinician Reviews - 23(3)
Page Number
11
Page Number
11
Publications
Publications
Topics
Article Type
Display Headline
Thumb: Scaling with Pitted Nail Plate
Display Headline
Thumb: Scaling with Pitted Nail Plate
Legacy Keywords
dermatology, scaling, thumb, hydrocortisone, cream, fungal infection, clotrimazole/betamethasone cream, terbinafine, stress, arthritis, scales, KOH prep, discoloration, Fungal infection, Eczema, Squamous cell carcinoma, Psoriasis
Legacy Keywords
dermatology, scaling, thumb, hydrocortisone, cream, fungal infection, clotrimazole/betamethasone cream, terbinafine, stress, arthritis, scales, KOH prep, discoloration, Fungal infection, Eczema, Squamous cell carcinoma, Psoriasis
Sections
Questionnaire Body

 

 

About a year ago, this 40-year-old man developed scaling on the distal one-third of his thumbnail. After an ini-tial apparent response to OTC hydrocortisone 1% cream, the condition began to worsen, spreading to more of the thumb. The patient’s primary care provider diagnosed fungal infection and prescribed a combination clotrima-zole/betamethasone cream, which had no effect. A subsequent four-month course of terbinafine (250 mg/d) also yielded no improvement. The patient then requested referral to dermatology. The patient considers himself “quite healthy,” aside from having moderately severe arthritis, for which he takes nabumetone (750 mg bid). His arthritis recently worsened, and he is scheduled to see a rheumatologist in two months. He admits to being under a great deal of stress shortly before his skin condition developed. As a result, he in-creased his alcohol intake for a few months, but he quit drinking altogether soon afterward. He denies any family history of skin disease and has no children or contact with animals. On examination, most of his thumb is covered by thick adherent white scales on a pinkish base. The margins are sharply defined. A KOH prep is performed, but no fungal elements are seen. The adjacent thumbnail has focal areas of pitting in the nail plate, as well as yellowish discoloration on the dis-tal edge. No such changes are seen on his other nails, and examination of his elbows, knees, trunk, and scalp fails to reveal any significant abnormalities.

 

Disallow All Ads

All of Her Friends Say She Has Ringworm

Article Type
Changed
Tue, 12/13/2016 - 12:08
Display Headline
All of Her Friends Say She Has Ringworm

ANSWER
The correct answer is pityriasis rosea (PR; choice “d”), which is commonly seen in patients ages 10 to 35 and is about twice as likely to occur in women as in men.

Lichen planus (LP; choice “a”) can mimic PR but lacks the peculiar centripetal scale and oval shape. Furthermore, it does not present with a herald patch.

Guttate psoriasis (choice “b”) could easily be confused for PR. However, it displays heavier white uniform scales with a salmon-pink base, tends to have a distinctly round configuration, and does not involve the appearance of a herald patch.

Secondary syphilis (choice “c”) can usually be ruled out by the sexual history, but also by the lack of a herald patch and the absence of centripetal scaling. Highly variable in appearance, the lesions of secondary syphilis are often seen on the palms.

DISCUSSION
PR was first described in 1860 by Camille Gibert, who used the term pityriasis to describe the fine scale seen with this condition, and chose the term rosea to denote the rosy or pink color.

For a variety of reasons, PR is assumed to be a viral exanthema since, as with many such eruptions, its incidence clusters in the fall and spring, it occurs in close contacts and families, and it is commonly seen in immunocompromised patients. In addition, acquiring the condition appears to confer lifelong immunity.

However, the jury is still out with regard to the exact virus responsible for the disease. Human herpesviruses 6 and 7 are the strongest candidates in terms of antibody production, but no herpesviral particles have been detected in tissue samples.

The so-called herald patch appears initially, in a majority of cases, as a salmon-pink patch that can become as large as 5 to 10 cm, on the trunk or arms. The smaller oval lesions begin to appear within a week or two, averaging 1 to 2 cm in diameter; most display the characteristic “centripetal” scaling, clearly sparing the lesions’ periphery and serving as an essentially pathognomic finding.

On darker-skinned patients, the lesions (including the herald patch) will tend to be brown to black. The examiner must then look for the other characteristic aspects of PR, including the oval (as opposed to round) shape, the long axes of which will often parallel the skin tension lines on the back to produce what is termed the “Christmas tree pattern.” In the author’s experience, the most consistent diagnostic finding is the centripetal scaling seen in at least a few lesions.

Since secondary syphilis is a major item in the differential, obtaining a careful sexual history is essential. If this is uncertain, or if the lesions are not a good fit for PR, obtaining a punch biopsy and serum rapid plasma reagin is necessary. The biopsy in secondary syphilis will show an infiltrate largely composed of plasma cells.

TREATMENT
Once the diagnosis of PR is made, patient education is essential. Affected patients should be reassured about the benign and self-limiting nature of the problem, but also about the likelihood that the condition will persist for up to nine weeks. Darker-skinned patients need to understand that the hyperpigmentation will last for months after the condition has resolved.

Relief of the itching experienced by 75% of PR patients can be achieved with topical steroids (eg, triamcinolone 0.1% cream) and oral antihistamines at bedtime (eg, hydroxyzine 25 to 50 mg) and/or during the daytime (cetirizine 10 mg/d), plus the liberal use of soothing OTC lotions (eg, those containing camphor and menthol). Systemic steroids appear to prolong the condition and are not terribly helpful in controlling the symptoms. In severe cases, phototherapy (narrow-band UVB) can be useful in controlling the itching.

Author and Disclosure Information

 

Joe R. Monroe, MPAS, PA

Issue
Clinician Reviews - 23(2)
Publications
Topics
Legacy Keywords
dermatology, lesion, ringworm, clotrimazole, Lichen planus, Guttate psoriasis, Secondary syphilis, Pityriasis rosea, pityriasis
Sections
Author and Disclosure Information

 

Joe R. Monroe, MPAS, PA

Author and Disclosure Information

 

Joe R. Monroe, MPAS, PA

ANSWER
The correct answer is pityriasis rosea (PR; choice “d”), which is commonly seen in patients ages 10 to 35 and is about twice as likely to occur in women as in men.

Lichen planus (LP; choice “a”) can mimic PR but lacks the peculiar centripetal scale and oval shape. Furthermore, it does not present with a herald patch.

Guttate psoriasis (choice “b”) could easily be confused for PR. However, it displays heavier white uniform scales with a salmon-pink base, tends to have a distinctly round configuration, and does not involve the appearance of a herald patch.

Secondary syphilis (choice “c”) can usually be ruled out by the sexual history, but also by the lack of a herald patch and the absence of centripetal scaling. Highly variable in appearance, the lesions of secondary syphilis are often seen on the palms.

DISCUSSION
PR was first described in 1860 by Camille Gibert, who used the term pityriasis to describe the fine scale seen with this condition, and chose the term rosea to denote the rosy or pink color.

For a variety of reasons, PR is assumed to be a viral exanthema since, as with many such eruptions, its incidence clusters in the fall and spring, it occurs in close contacts and families, and it is commonly seen in immunocompromised patients. In addition, acquiring the condition appears to confer lifelong immunity.

However, the jury is still out with regard to the exact virus responsible for the disease. Human herpesviruses 6 and 7 are the strongest candidates in terms of antibody production, but no herpesviral particles have been detected in tissue samples.

The so-called herald patch appears initially, in a majority of cases, as a salmon-pink patch that can become as large as 5 to 10 cm, on the trunk or arms. The smaller oval lesions begin to appear within a week or two, averaging 1 to 2 cm in diameter; most display the characteristic “centripetal” scaling, clearly sparing the lesions’ periphery and serving as an essentially pathognomic finding.

On darker-skinned patients, the lesions (including the herald patch) will tend to be brown to black. The examiner must then look for the other characteristic aspects of PR, including the oval (as opposed to round) shape, the long axes of which will often parallel the skin tension lines on the back to produce what is termed the “Christmas tree pattern.” In the author’s experience, the most consistent diagnostic finding is the centripetal scaling seen in at least a few lesions.

Since secondary syphilis is a major item in the differential, obtaining a careful sexual history is essential. If this is uncertain, or if the lesions are not a good fit for PR, obtaining a punch biopsy and serum rapid plasma reagin is necessary. The biopsy in secondary syphilis will show an infiltrate largely composed of plasma cells.

TREATMENT
Once the diagnosis of PR is made, patient education is essential. Affected patients should be reassured about the benign and self-limiting nature of the problem, but also about the likelihood that the condition will persist for up to nine weeks. Darker-skinned patients need to understand that the hyperpigmentation will last for months after the condition has resolved.

Relief of the itching experienced by 75% of PR patients can be achieved with topical steroids (eg, triamcinolone 0.1% cream) and oral antihistamines at bedtime (eg, hydroxyzine 25 to 50 mg) and/or during the daytime (cetirizine 10 mg/d), plus the liberal use of soothing OTC lotions (eg, those containing camphor and menthol). Systemic steroids appear to prolong the condition and are not terribly helpful in controlling the symptoms. In severe cases, phototherapy (narrow-band UVB) can be useful in controlling the itching.

ANSWER
The correct answer is pityriasis rosea (PR; choice “d”), which is commonly seen in patients ages 10 to 35 and is about twice as likely to occur in women as in men.

Lichen planus (LP; choice “a”) can mimic PR but lacks the peculiar centripetal scale and oval shape. Furthermore, it does not present with a herald patch.

Guttate psoriasis (choice “b”) could easily be confused for PR. However, it displays heavier white uniform scales with a salmon-pink base, tends to have a distinctly round configuration, and does not involve the appearance of a herald patch.

Secondary syphilis (choice “c”) can usually be ruled out by the sexual history, but also by the lack of a herald patch and the absence of centripetal scaling. Highly variable in appearance, the lesions of secondary syphilis are often seen on the palms.

DISCUSSION
PR was first described in 1860 by Camille Gibert, who used the term pityriasis to describe the fine scale seen with this condition, and chose the term rosea to denote the rosy or pink color.

For a variety of reasons, PR is assumed to be a viral exanthema since, as with many such eruptions, its incidence clusters in the fall and spring, it occurs in close contacts and families, and it is commonly seen in immunocompromised patients. In addition, acquiring the condition appears to confer lifelong immunity.

However, the jury is still out with regard to the exact virus responsible for the disease. Human herpesviruses 6 and 7 are the strongest candidates in terms of antibody production, but no herpesviral particles have been detected in tissue samples.

The so-called herald patch appears initially, in a majority of cases, as a salmon-pink patch that can become as large as 5 to 10 cm, on the trunk or arms. The smaller oval lesions begin to appear within a week or two, averaging 1 to 2 cm in diameter; most display the characteristic “centripetal” scaling, clearly sparing the lesions’ periphery and serving as an essentially pathognomic finding.

On darker-skinned patients, the lesions (including the herald patch) will tend to be brown to black. The examiner must then look for the other characteristic aspects of PR, including the oval (as opposed to round) shape, the long axes of which will often parallel the skin tension lines on the back to produce what is termed the “Christmas tree pattern.” In the author’s experience, the most consistent diagnostic finding is the centripetal scaling seen in at least a few lesions.

Since secondary syphilis is a major item in the differential, obtaining a careful sexual history is essential. If this is uncertain, or if the lesions are not a good fit for PR, obtaining a punch biopsy and serum rapid plasma reagin is necessary. The biopsy in secondary syphilis will show an infiltrate largely composed of plasma cells.

TREATMENT
Once the diagnosis of PR is made, patient education is essential. Affected patients should be reassured about the benign and self-limiting nature of the problem, but also about the likelihood that the condition will persist for up to nine weeks. Darker-skinned patients need to understand that the hyperpigmentation will last for months after the condition has resolved.

Relief of the itching experienced by 75% of PR patients can be achieved with topical steroids (eg, triamcinolone 0.1% cream) and oral antihistamines at bedtime (eg, hydroxyzine 25 to 50 mg) and/or during the daytime (cetirizine 10 mg/d), plus the liberal use of soothing OTC lotions (eg, those containing camphor and menthol). Systemic steroids appear to prolong the condition and are not terribly helpful in controlling the symptoms. In severe cases, phototherapy (narrow-band UVB) can be useful in controlling the itching.

Issue
Clinician Reviews - 23(2)
Issue
Clinician Reviews - 23(2)
Publications
Publications
Topics
Article Type
Display Headline
All of Her Friends Say She Has Ringworm
Display Headline
All of Her Friends Say She Has Ringworm
Legacy Keywords
dermatology, lesion, ringworm, clotrimazole, Lichen planus, Guttate psoriasis, Secondary syphilis, Pityriasis rosea, pityriasis
Legacy Keywords
dermatology, lesion, ringworm, clotrimazole, Lichen planus, Guttate psoriasis, Secondary syphilis, Pityriasis rosea, pityriasis
Sections
Questionnaire Body

 

 

Three weeks ago, a 25-year-old woman noticed an asymp¬tomatic lesion of unknown origin on her chest. Since then, smaller versions have appeared in “crops” on her trunk, arms, and lower neck. Friends were unanimous in their opinion that she had “ringworm,” so she consulted her pharmacist, who recommended clotrimazole cream. Despite her use of it, however, the lesions continue to increase in number. Her original lesion has become less red and scaly, though. The patient has felt fine from the outset and maintains that she is “quite healthy” in other respects. Employed as an IT technician, she denies any exposure to children, pets, or sexually transmitted diseases. The patient, who is African-American, has type V skin. Her original lesion—located on her left inframammary chest—is dark brown, macular, oval to round, and measures about 3.8 cm. On her trunk, arms, and lower neck, 15 to 20 oval, papulosquamous lesions are seen; these are widely scattered, all hyperpigmented (brown), and average 1.5 cm in diameter. Several of these smaller lesions have scaly centers that spare the peripheral margins. The long axes of her oval back lesions are parallel with natural lines of cleavage in the skin.

 

Disallow All Ads

Why This Child Hates to Put On Socks

Article Type
Changed
Tue, 12/13/2016 - 12:08
Display Headline
Why This Child Hates to Put On Socks

ANSWER


The correct answer is juvenile plantar dermatosis (JPD; choice “b”). It is a condition related to having thin, dry, hyperreactive skin exposed to friction, wetting and drying, and constant exposure to the nonpermeable surfaces of shoes.
Pitted keratolysis (choice “a”) is a condition caused by sweating and increased warmth. The plantar keratin is broken down with the help of bacteria that overgrow in affected areas; this eventuates in focal loss of keratin in arcuate patterns. It is quite unlikely to occur prior to puberty.

Tinea pedis (choice “c”) is dermatophytosis, or fungal infection of the foot. It is also unusual prior to puberty, unlikely to present in the manner seen in this case, and likely to have responded at least partially to antifungal creams.
Psoriasis (choice “d”) seldom presents with fissuring, would not be confined to weight-bearing surfaces, and would probably have involved other areas, such as the scalp, elbows, knees, or nails.

DISCUSSION


JPD, also known as juvenile plantar dermatitis, is found almost exclusively on the weight-bearing surfaces of the feet of children ages 4 to 8—mostly boys, for whom this represents a manifestation of the atopic diathesis. Seen mostly in the summer, it is thought to be triggered by friction, wetting and drying, and shoe selection (ie, plastic rather than leather soles).

Affected children not only have dry, sensitive skin; their skin is actually thin and fragile as well. Plastic or other synthetic shoe surfaces worn in the summertime are thought to contribute to the friction, heat, and sweating necessary to produce these changes.

As in this case, JPD is often mistaken for tinea pedis but has nothing to do with infection of any kind. Tinea pedis is uncommon in children this young, and it would present in completely different ways, such as between the toes (especially the fourth and fifth) or with blisters on the instep.
Psoriasis, though not unknown in this age-group, does not resemble JPD clinically at all. When suspected, the diagnosis of psoriasis can be corroborated by finding it elsewhere (eg, through a positive family history or biopsy).

Pitted keratolyis is common enough, but is seen in older teens and men whose feet are prone to sweat a great deal. The choice of shoes and occupation are often crucial factors in its development. The clinical hallmark is arcuate whitish maceration on weight-bearing surfaces, which are often malodorous as well.

TREATMENT

The first treatment for JPD is education of parents and patients, reassuring them about the relatively benign nature of the problem. Moisturizing frequently with petrolatum-based moisturizers is necessary for prevention, but changing the type of shoes worn is the most effective step to take; it is also the most difficult, since children this age favor cheap, plastic flip-flops or shoes in the summer.

For the fissures, spraying on a flexible spray bandage can be helpful in protecting them and allowing them to heal. With significant inflammation, the use of mild steroid ointments, such as hydrocortisone 2.5%, can help. But by far, the best relief comes with the change in season and the choice of shoe (leather-soled).

Author and Disclosure Information

 

Joe R. Monroe, MPAS, PA

Issue
Clinician Reviews - 23(1)
Publications
Topics
Page Number
7
Legacy Keywords
athlete's foot, dermatology, JPD, Pitted keratolysis, Juvenile plantar dermatosis, Tinea pedis, Psoriasis, child, atopic diathesis
Sections
Author and Disclosure Information

 

Joe R. Monroe, MPAS, PA

Author and Disclosure Information

 

Joe R. Monroe, MPAS, PA

ANSWER


The correct answer is juvenile plantar dermatosis (JPD; choice “b”). It is a condition related to having thin, dry, hyperreactive skin exposed to friction, wetting and drying, and constant exposure to the nonpermeable surfaces of shoes.
Pitted keratolysis (choice “a”) is a condition caused by sweating and increased warmth. The plantar keratin is broken down with the help of bacteria that overgrow in affected areas; this eventuates in focal loss of keratin in arcuate patterns. It is quite unlikely to occur prior to puberty.

Tinea pedis (choice “c”) is dermatophytosis, or fungal infection of the foot. It is also unusual prior to puberty, unlikely to present in the manner seen in this case, and likely to have responded at least partially to antifungal creams.
Psoriasis (choice “d”) seldom presents with fissuring, would not be confined to weight-bearing surfaces, and would probably have involved other areas, such as the scalp, elbows, knees, or nails.

DISCUSSION


JPD, also known as juvenile plantar dermatitis, is found almost exclusively on the weight-bearing surfaces of the feet of children ages 4 to 8—mostly boys, for whom this represents a manifestation of the atopic diathesis. Seen mostly in the summer, it is thought to be triggered by friction, wetting and drying, and shoe selection (ie, plastic rather than leather soles).

Affected children not only have dry, sensitive skin; their skin is actually thin and fragile as well. Plastic or other synthetic shoe surfaces worn in the summertime are thought to contribute to the friction, heat, and sweating necessary to produce these changes.

As in this case, JPD is often mistaken for tinea pedis but has nothing to do with infection of any kind. Tinea pedis is uncommon in children this young, and it would present in completely different ways, such as between the toes (especially the fourth and fifth) or with blisters on the instep.
Psoriasis, though not unknown in this age-group, does not resemble JPD clinically at all. When suspected, the diagnosis of psoriasis can be corroborated by finding it elsewhere (eg, through a positive family history or biopsy).

Pitted keratolyis is common enough, but is seen in older teens and men whose feet are prone to sweat a great deal. The choice of shoes and occupation are often crucial factors in its development. The clinical hallmark is arcuate whitish maceration on weight-bearing surfaces, which are often malodorous as well.

TREATMENT

The first treatment for JPD is education of parents and patients, reassuring them about the relatively benign nature of the problem. Moisturizing frequently with petrolatum-based moisturizers is necessary for prevention, but changing the type of shoes worn is the most effective step to take; it is also the most difficult, since children this age favor cheap, plastic flip-flops or shoes in the summer.

For the fissures, spraying on a flexible spray bandage can be helpful in protecting them and allowing them to heal. With significant inflammation, the use of mild steroid ointments, such as hydrocortisone 2.5%, can help. But by far, the best relief comes with the change in season and the choice of shoe (leather-soled).

ANSWER


The correct answer is juvenile plantar dermatosis (JPD; choice “b”). It is a condition related to having thin, dry, hyperreactive skin exposed to friction, wetting and drying, and constant exposure to the nonpermeable surfaces of shoes.
Pitted keratolysis (choice “a”) is a condition caused by sweating and increased warmth. The plantar keratin is broken down with the help of bacteria that overgrow in affected areas; this eventuates in focal loss of keratin in arcuate patterns. It is quite unlikely to occur prior to puberty.

Tinea pedis (choice “c”) is dermatophytosis, or fungal infection of the foot. It is also unusual prior to puberty, unlikely to present in the manner seen in this case, and likely to have responded at least partially to antifungal creams.
Psoriasis (choice “d”) seldom presents with fissuring, would not be confined to weight-bearing surfaces, and would probably have involved other areas, such as the scalp, elbows, knees, or nails.

DISCUSSION


JPD, also known as juvenile plantar dermatitis, is found almost exclusively on the weight-bearing surfaces of the feet of children ages 4 to 8—mostly boys, for whom this represents a manifestation of the atopic diathesis. Seen mostly in the summer, it is thought to be triggered by friction, wetting and drying, and shoe selection (ie, plastic rather than leather soles).

Affected children not only have dry, sensitive skin; their skin is actually thin and fragile as well. Plastic or other synthetic shoe surfaces worn in the summertime are thought to contribute to the friction, heat, and sweating necessary to produce these changes.

As in this case, JPD is often mistaken for tinea pedis but has nothing to do with infection of any kind. Tinea pedis is uncommon in children this young, and it would present in completely different ways, such as between the toes (especially the fourth and fifth) or with blisters on the instep.
Psoriasis, though not unknown in this age-group, does not resemble JPD clinically at all. When suspected, the diagnosis of psoriasis can be corroborated by finding it elsewhere (eg, through a positive family history or biopsy).

Pitted keratolyis is common enough, but is seen in older teens and men whose feet are prone to sweat a great deal. The choice of shoes and occupation are often crucial factors in its development. The clinical hallmark is arcuate whitish maceration on weight-bearing surfaces, which are often malodorous as well.

TREATMENT

The first treatment for JPD is education of parents and patients, reassuring them about the relatively benign nature of the problem. Moisturizing frequently with petrolatum-based moisturizers is necessary for prevention, but changing the type of shoes worn is the most effective step to take; it is also the most difficult, since children this age favor cheap, plastic flip-flops or shoes in the summer.

For the fissures, spraying on a flexible spray bandage can be helpful in protecting them and allowing them to heal. With significant inflammation, the use of mild steroid ointments, such as hydrocortisone 2.5%, can help. But by far, the best relief comes with the change in season and the choice of shoe (leather-soled).

Issue
Clinician Reviews - 23(1)
Issue
Clinician Reviews - 23(1)
Page Number
7
Page Number
7
Publications
Publications
Topics
Article Type
Display Headline
Why This Child Hates to Put On Socks
Display Headline
Why This Child Hates to Put On Socks
Legacy Keywords
athlete's foot, dermatology, JPD, Pitted keratolysis, Juvenile plantar dermatosis, Tinea pedis, Psoriasis, child, atopic diathesis
Legacy Keywords
athlete's foot, dermatology, JPD, Pitted keratolysis, Juvenile plantar dermatosis, Tinea pedis, Psoriasis, child, atopic diathesis
Sections
Questionnaire Body

 

 

The distraught mother of an 8-year-old boy brings him urgently to dermatology for evaluation of a condition that has affected his feet for the past two summers. Convinced he has “caught” athlete’s foot, she tried several OTC antifungal creams and sprays, with no good effect. The patient denies symptoms except occasional stinging. In his view, the biggest problem is that the bottoms of his feet are so rough that he hates to put on socks. Additional history taking reveals that the child is markedly atopic, with seasonal allergies, asthma, dry, sensitive skin, and eczema. As an infant, his diaper rashes were so severe that he was hospitalized twice. On inspection, the weight-bearing surfaces of both feet are fissured and shiny, with modest inflammation evident. The plantar aspects of both big toes are especially affected. Though these areas are rough and dry, there is no edema, increased warmth, or tenderness on palpation. His skin elsewhere, though dry, is free of obvious lesions.

 

Disallow All Ads

Edematous Changes Coincide with New Job

Article Type
Changed
Tue, 12/13/2016 - 12:08
Display Headline
Edematous Changes Coincide with New Job

ANSWER
The correct answer is gram-negative bacteria (choice “d”); Pseudomonas is the most likely culprit. Candida albicans (choice “a”), a yeast, is an unlikely cause of this problem and even more unlikely to show up on a bacterial culture. Coagulase-positive staph aureus (choice “b”) is typically associated with infections involving the acute onset of redness, pain, swelling, and pus formation, not the indolent, chronic, low-grade process seen in this case. Trichophyton rubrum (choice “c”) is a dermatophyte, the most common fungal cause of athlete’s feet. The bacterial culture could not have grown a dermatophyte, which needs special media and conditions to grow.

DISCUSSSION
Gram-negative interweb impetigo is a relatively common dermatologic entity, which can be caused by any number of organisms found in fecal material. Pseudomonas, Klebsiella, Proteus, and Acinetobacter are among the more common culprits. These types of infections tend to be much more indolent than the more common staph- and strep-caused cellulitis, which are more likely to create acute redness, swelling, pain, and pus. 

Both types of bacterial infections need certain conditions in order to develop. These include excessive heat, sweat, and perhaps most significantly, a break in the skin barrier. Ironically, these fissures are often caused by dermatophytes, in the form of tinea pedis, which is, of course, far better known for causing rashes of the foot.

But tinea pedis is more likely to be found between the third and fourth or the fourth and fifth toes. It creates itching and maceration but rarely causes diffuse redness or edema, and even more rarely leads to pain (unless there is a secondary bacterial infection). As mentioned, given the indolence of this infective process, a culture result showing staph or strep was unlikely.

The culture in this case showed Proteus, for which the minocycline was predictably effective. The rationale for obtaining the acid-fast culture was the possibility of finding Mycobacteria species such as M fortuitum, which is known to cause chronic indolent infections in feet and legs. These, however, more typically manifest with solitary eroded or ulcerated lesions. (Minocycline would have been effective against this organism.)

The use of the topical econazole served two purposes: While this was clearly not classic tinea pedis, it was still possible a dermatophyte or a yeast could have played a role in the creation of the initial fissuring; econazole will help control this, long term. Econazole also has significant antibacterial action and is particularly useful to help prevent future flares.          

Author and Disclosure Information

 

Joe R. Monroe, MPAS, PA

Issue
Clinician Reviews - 22(12)
Publications
Topics
Page Number
11
Legacy Keywords
redness, swelling, pain, interdigital, web, feet, terbinafine, dorsal, edematous changes, Candida albicans, Coagulase-positive Staphylococcus aureus, Trichophyton rubrum, Gram-negative bacteria, Pseudomonas, Proteus
Sections
Author and Disclosure Information

 

Joe R. Monroe, MPAS, PA

Author and Disclosure Information

 

Joe R. Monroe, MPAS, PA

ANSWER
The correct answer is gram-negative bacteria (choice “d”); Pseudomonas is the most likely culprit. Candida albicans (choice “a”), a yeast, is an unlikely cause of this problem and even more unlikely to show up on a bacterial culture. Coagulase-positive staph aureus (choice “b”) is typically associated with infections involving the acute onset of redness, pain, swelling, and pus formation, not the indolent, chronic, low-grade process seen in this case. Trichophyton rubrum (choice “c”) is a dermatophyte, the most common fungal cause of athlete’s feet. The bacterial culture could not have grown a dermatophyte, which needs special media and conditions to grow.

DISCUSSSION
Gram-negative interweb impetigo is a relatively common dermatologic entity, which can be caused by any number of organisms found in fecal material. Pseudomonas, Klebsiella, Proteus, and Acinetobacter are among the more common culprits. These types of infections tend to be much more indolent than the more common staph- and strep-caused cellulitis, which are more likely to create acute redness, swelling, pain, and pus. 

Both types of bacterial infections need certain conditions in order to develop. These include excessive heat, sweat, and perhaps most significantly, a break in the skin barrier. Ironically, these fissures are often caused by dermatophytes, in the form of tinea pedis, which is, of course, far better known for causing rashes of the foot.

But tinea pedis is more likely to be found between the third and fourth or the fourth and fifth toes. It creates itching and maceration but rarely causes diffuse redness or edema, and even more rarely leads to pain (unless there is a secondary bacterial infection). As mentioned, given the indolence of this infective process, a culture result showing staph or strep was unlikely.

The culture in this case showed Proteus, for which the minocycline was predictably effective. The rationale for obtaining the acid-fast culture was the possibility of finding Mycobacteria species such as M fortuitum, which is known to cause chronic indolent infections in feet and legs. These, however, more typically manifest with solitary eroded or ulcerated lesions. (Minocycline would have been effective against this organism.)

The use of the topical econazole served two purposes: While this was clearly not classic tinea pedis, it was still possible a dermatophyte or a yeast could have played a role in the creation of the initial fissuring; econazole will help control this, long term. Econazole also has significant antibacterial action and is particularly useful to help prevent future flares.          

ANSWER
The correct answer is gram-negative bacteria (choice “d”); Pseudomonas is the most likely culprit. Candida albicans (choice “a”), a yeast, is an unlikely cause of this problem and even more unlikely to show up on a bacterial culture. Coagulase-positive staph aureus (choice “b”) is typically associated with infections involving the acute onset of redness, pain, swelling, and pus formation, not the indolent, chronic, low-grade process seen in this case. Trichophyton rubrum (choice “c”) is a dermatophyte, the most common fungal cause of athlete’s feet. The bacterial culture could not have grown a dermatophyte, which needs special media and conditions to grow.

DISCUSSSION
Gram-negative interweb impetigo is a relatively common dermatologic entity, which can be caused by any number of organisms found in fecal material. Pseudomonas, Klebsiella, Proteus, and Acinetobacter are among the more common culprits. These types of infections tend to be much more indolent than the more common staph- and strep-caused cellulitis, which are more likely to create acute redness, swelling, pain, and pus. 

Both types of bacterial infections need certain conditions in order to develop. These include excessive heat, sweat, and perhaps most significantly, a break in the skin barrier. Ironically, these fissures are often caused by dermatophytes, in the form of tinea pedis, which is, of course, far better known for causing rashes of the foot.

But tinea pedis is more likely to be found between the third and fourth or the fourth and fifth toes. It creates itching and maceration but rarely causes diffuse redness or edema, and even more rarely leads to pain (unless there is a secondary bacterial infection). As mentioned, given the indolence of this infective process, a culture result showing staph or strep was unlikely.

The culture in this case showed Proteus, for which the minocycline was predictably effective. The rationale for obtaining the acid-fast culture was the possibility of finding Mycobacteria species such as M fortuitum, which is known to cause chronic indolent infections in feet and legs. These, however, more typically manifest with solitary eroded or ulcerated lesions. (Minocycline would have been effective against this organism.)

The use of the topical econazole served two purposes: While this was clearly not classic tinea pedis, it was still possible a dermatophyte or a yeast could have played a role in the creation of the initial fissuring; econazole will help control this, long term. Econazole also has significant antibacterial action and is particularly useful to help prevent future flares.          

Issue
Clinician Reviews - 22(12)
Issue
Clinician Reviews - 22(12)
Page Number
11
Page Number
11
Publications
Publications
Topics
Article Type
Display Headline
Edematous Changes Coincide with New Job
Display Headline
Edematous Changes Coincide with New Job
Legacy Keywords
redness, swelling, pain, interdigital, web, feet, terbinafine, dorsal, edematous changes, Candida albicans, Coagulase-positive Staphylococcus aureus, Trichophyton rubrum, Gram-negative bacteria, Pseudomonas, Proteus
Legacy Keywords
redness, swelling, pain, interdigital, web, feet, terbinafine, dorsal, edematous changes, Candida albicans, Coagulase-positive Staphylococcus aureus, Trichophyton rubrum, Gram-negative bacteria, Pseudomonas, Proteus
Sections
Questionnaire Body

 

 

A 50-year-old man presents with a six-month history of worsening redness, swelling, and pain in the interdigital web spaces of both feet. Numerous treatments—most recently, a two-month course of terbinafine 250 mg/d—have not induced any change. The problem manifested as mild cracking between the first and second toes, dorsal aspect, and slowly spread laterally to involve all four web spaces. This pro-cess coincided with the start of a new job in which the patient is on his feet, wearing steel-toed boots, for 12 hours per day in a hot environment. Assuming the problem was athlete’s foot, he tried OTC clotrimazole and terbinafine creams; neither helped at all. In fact, the patient’s pain is now so bad that he has difficult walking. There is no history of smoking, diabetes, or other serious health problems. Examination shows distinct demarcated, dusky-red, edematous changes largely confined to the dorsal aspect. The actual deep interdigital space and the volar aspects of these areas are spared. Slight epidermal fissuring is seen on the dorsal aspect of each web space, from which a small amount of fluid can be coaxed. The fluid is sent for bacterial and acid-fast cultures. In the meantime, the patient is prescribed oral minocycline 100 bid and topical econazole cream and in-structed to return in two weeks. At that time, his condition is almost completely resolved.

 

Disallow All Ads

Topical Steroids: the Solution or the Cause?

Article Type
Changed
Tue, 12/13/2016 - 12:08
Display Headline
Topical Steroids: the Solution or the Cause?

ANSWER
The correct answer is all of the above (choice “d”). Prolonged injudicious use of topical steroids can cause a number of problems, including these; they are collectively termed iatrogenic since they are ultimately caused by prescribed medication. One of the more difficult aspects of this problem to deal with is the “addictive” state, in which withdrawal symptoms compel the patient to continue applying the offending steroid cream.

DISCUSSION
This is a relatively common scenario in dermatology offices. The misuse of topical steroids is well known, and something we strive to prevent—but with mixed results. It’s one of the reasons we’re stingy with refills of such medications, requiring the patient to be seen at least once a year. Unfortunately, this patient had been getting “refills” from friends in Mexico; patients often “borrow” steroid creams from household members or friends, or use products prescribed for one condition to treat others for which they were not intended.

The primary mode of action of topical steroids is vasoconstriction, a positive thing in terms of reduction of inflammation. The bad news is that continuous use of class 1 (the most powerful) steroids, such as clobetasol, can cause such profound and prolonged vasoconstriction that the skin effectively loses its blood supply and withers, sometimes down to adipose tissue. As one might suspect, this is more likely in already thin-skinned areas, including the antecubital area, face, neck, eyelids, and genitals, where the creation of striae is especially common.

Fairly early on in this process, before frank atrophy occurs, the condition being treated usually resolves. However, when the steroid is stopped, stinging and itching immediately return—which, of course, causes the patient to reapply the medication, perpetuating the vicious cycle.

The cycle is ultimately broken by gradual reduction in the frequency of application of successively weaker steroids. Usually, the skin gradually regenerates and returns to normal. In this case, the process will be lengthy and will almost certainly result in significant scarring.

Even injudicious application of weaker classes of steroids (eg, hydrocortisone 2.5% cream) to areas such as the face can result in a range of deleterious effects, including localized rosacea-like eruption or erythema. It has been reported that approximately 75% of cases of perioral dermatitis are either caused by or exacerbated by the application of topical steroids.

Topical application of even mid-strength steroids can also have systemic effects (eg, adrenal suppression, hyperglycemia) if applied over large areas. This is especially true when pediatric patients are involved.

Prevention of these iatrogenic effects lies in selecting the lowest strength steroid for the condition and area in question, then using them sparingly: no more than twice a day, and for no more than five days in a row, stopping for two consecutive days to allow the skin to regenerate. Even more caution should be exercised in treating children and when applying the product to intertriginous areas (skin-on-skin areas, such as the groin, in axillae, or under the breasts). Covering steroid-treated areas with anything—bandages, socks, even skin—effectively potentiates the positive and negative effects of steroids.

Author and Disclosure Information

 

Joe R. Monroe, MPAS, PA

Issue
Clinician Reviews - 22(11)
Publications
Topics
Page Number
1
Legacy Keywords
dermatology, antecuital area, clobetasol, eczema, epidermal atrophy, purpura, subdermal vasculature, Steroid addiction, Steroid atrophy, Iatrogenic side effect
Sections
Author and Disclosure Information

 

Joe R. Monroe, MPAS, PA

Author and Disclosure Information

 

Joe R. Monroe, MPAS, PA

ANSWER
The correct answer is all of the above (choice “d”). Prolonged injudicious use of topical steroids can cause a number of problems, including these; they are collectively termed iatrogenic since they are ultimately caused by prescribed medication. One of the more difficult aspects of this problem to deal with is the “addictive” state, in which withdrawal symptoms compel the patient to continue applying the offending steroid cream.

DISCUSSION
This is a relatively common scenario in dermatology offices. The misuse of topical steroids is well known, and something we strive to prevent—but with mixed results. It’s one of the reasons we’re stingy with refills of such medications, requiring the patient to be seen at least once a year. Unfortunately, this patient had been getting “refills” from friends in Mexico; patients often “borrow” steroid creams from household members or friends, or use products prescribed for one condition to treat others for which they were not intended.

The primary mode of action of topical steroids is vasoconstriction, a positive thing in terms of reduction of inflammation. The bad news is that continuous use of class 1 (the most powerful) steroids, such as clobetasol, can cause such profound and prolonged vasoconstriction that the skin effectively loses its blood supply and withers, sometimes down to adipose tissue. As one might suspect, this is more likely in already thin-skinned areas, including the antecubital area, face, neck, eyelids, and genitals, where the creation of striae is especially common.

Fairly early on in this process, before frank atrophy occurs, the condition being treated usually resolves. However, when the steroid is stopped, stinging and itching immediately return—which, of course, causes the patient to reapply the medication, perpetuating the vicious cycle.

The cycle is ultimately broken by gradual reduction in the frequency of application of successively weaker steroids. Usually, the skin gradually regenerates and returns to normal. In this case, the process will be lengthy and will almost certainly result in significant scarring.

Even injudicious application of weaker classes of steroids (eg, hydrocortisone 2.5% cream) to areas such as the face can result in a range of deleterious effects, including localized rosacea-like eruption or erythema. It has been reported that approximately 75% of cases of perioral dermatitis are either caused by or exacerbated by the application of topical steroids.

Topical application of even mid-strength steroids can also have systemic effects (eg, adrenal suppression, hyperglycemia) if applied over large areas. This is especially true when pediatric patients are involved.

Prevention of these iatrogenic effects lies in selecting the lowest strength steroid for the condition and area in question, then using them sparingly: no more than twice a day, and for no more than five days in a row, stopping for two consecutive days to allow the skin to regenerate. Even more caution should be exercised in treating children and when applying the product to intertriginous areas (skin-on-skin areas, such as the groin, in axillae, or under the breasts). Covering steroid-treated areas with anything—bandages, socks, even skin—effectively potentiates the positive and negative effects of steroids.

ANSWER
The correct answer is all of the above (choice “d”). Prolonged injudicious use of topical steroids can cause a number of problems, including these; they are collectively termed iatrogenic since they are ultimately caused by prescribed medication. One of the more difficult aspects of this problem to deal with is the “addictive” state, in which withdrawal symptoms compel the patient to continue applying the offending steroid cream.

DISCUSSION
This is a relatively common scenario in dermatology offices. The misuse of topical steroids is well known, and something we strive to prevent—but with mixed results. It’s one of the reasons we’re stingy with refills of such medications, requiring the patient to be seen at least once a year. Unfortunately, this patient had been getting “refills” from friends in Mexico; patients often “borrow” steroid creams from household members or friends, or use products prescribed for one condition to treat others for which they were not intended.

The primary mode of action of topical steroids is vasoconstriction, a positive thing in terms of reduction of inflammation. The bad news is that continuous use of class 1 (the most powerful) steroids, such as clobetasol, can cause such profound and prolonged vasoconstriction that the skin effectively loses its blood supply and withers, sometimes down to adipose tissue. As one might suspect, this is more likely in already thin-skinned areas, including the antecubital area, face, neck, eyelids, and genitals, where the creation of striae is especially common.

Fairly early on in this process, before frank atrophy occurs, the condition being treated usually resolves. However, when the steroid is stopped, stinging and itching immediately return—which, of course, causes the patient to reapply the medication, perpetuating the vicious cycle.

The cycle is ultimately broken by gradual reduction in the frequency of application of successively weaker steroids. Usually, the skin gradually regenerates and returns to normal. In this case, the process will be lengthy and will almost certainly result in significant scarring.

Even injudicious application of weaker classes of steroids (eg, hydrocortisone 2.5% cream) to areas such as the face can result in a range of deleterious effects, including localized rosacea-like eruption or erythema. It has been reported that approximately 75% of cases of perioral dermatitis are either caused by or exacerbated by the application of topical steroids.

Topical application of even mid-strength steroids can also have systemic effects (eg, adrenal suppression, hyperglycemia) if applied over large areas. This is especially true when pediatric patients are involved.

Prevention of these iatrogenic effects lies in selecting the lowest strength steroid for the condition and area in question, then using them sparingly: no more than twice a day, and for no more than five days in a row, stopping for two consecutive days to allow the skin to regenerate. Even more caution should be exercised in treating children and when applying the product to intertriginous areas (skin-on-skin areas, such as the groin, in axillae, or under the breasts). Covering steroid-treated areas with anything—bandages, socks, even skin—effectively potentiates the positive and negative effects of steroids.

Issue
Clinician Reviews - 22(11)
Issue
Clinician Reviews - 22(11)
Page Number
1
Page Number
1
Publications
Publications
Topics
Article Type
Display Headline
Topical Steroids: the Solution or the Cause?
Display Headline
Topical Steroids: the Solution or the Cause?
Legacy Keywords
dermatology, antecuital area, clobetasol, eczema, epidermal atrophy, purpura, subdermal vasculature, Steroid addiction, Steroid atrophy, Iatrogenic side effect
Legacy Keywords
dermatology, antecuital area, clobetasol, eczema, epidermal atrophy, purpura, subdermal vasculature, Steroid addiction, Steroid atrophy, Iatrogenic side effect
Sections
Questionnaire Body

 

 

A 59-year-old man presents with skin changes on both antecubital areas. For more than a year, he has applied clobetasol 0.05% cream at least twice daily to the area ostensibly for treatment of long-standing eczema, which has affected not only the antecubital areas but also the patient’s legs. In addition to the eczema, he has a history of atopy, marked by seasonal allergies and asthma. He notes that his stress level has increased in the past several months, which he suspects has contributed to his itching. On examination, marked epidermal atrophy is seen in both antecubital areas, along with extensive purpura. Surface adnexal structures, such as hair, follicles, and skin lines, are sparse at best, but dermal and subdermal vasculature are readily visible. In the midst of the affected area on the right arm, a nickel-sized, full-thickness defect is noted. Beneath it, adipose tissue can be seen. Clearly, these changes are attributable to the effects of the clobetasol, which the patient is advised to stop. But he replies that when he does, the treated areas burn and itch even more, until he obtains relief by applying more clobetasol.

 

Disallow All Ads

Is Leprosy the Cause of This Girl's Lesion?

Article Type
Changed
Tue, 12/13/2016 - 12:08
Display Headline
Is Leprosy the Cause of This Girl's Lesion?

ANSWER
The correct answer is postinflammatory hypopigmentation (choice “c”), in this case secondary to eczema in a classic antecubital location. Leprosy (choice “a”) is more common than one might imagine, but it does not appear overnight and does not involve overt inflammation. Vitiligo (choice “b”) does not appear suddenly and rarely involves the type of inflammation seen in this case. Lichen sclerosis et atrophicus (choice “d”) is an inflammatory condition that presents with hypopigmentation and epidermal atrophy; however, it is gradual in onset and would not exhibit papulosquamous inflammation.

DISCUSSION
The more color in the skin, the more the loss of that color stands out. Patients and families with darker skin are often understandably upset by the contrast. Providers need a differential for pigment loss, including the items mentioned—some of which have the potential to be dreadfully serious.

Two relevant facts stand out in this case: the rapidity of onset and the history of eczema, in which secondary pigment loss can occur. As mentioned, it is especially obvious in those with darker skin. Fortunately, once the eczema calms down, the hypopigmentation resolves and normal color returns.

Paradoxically, it’s not at all unusual to see postinflammatory hyperpigmentation, especially in those with skin of types IV and V (eg, African-Americans, some Hispanics, and those of Indian ancestry). Eczema is a common cause, but the inflammation can be from almost any source, including trauma, burns, or even acne.

Had this patient’s diagnosis not been obvious, a biopsy might have been indicated due to the serious nature of some of the items in the differential. Vitiligo, for example, can be very disfiguring, especially on a dark-skinned individual. It tends to become widespread and permanent, unless it’s caught and treated early on. Other conditions involving hypopigmentation include sarcoidosis, lupus, and morphea.

All of these conditions are unusual, if not rare, compared with atopic dermatitis (AD), which this patient has. AD is so common that almost 20% of newborns develop it. Eczema is one of the more typical manifestations, along with dry, sensitive skin, seasonal allergies, and reactive airway disease. Corroboration of the diagnosis is usually easily accomplished by taking a family history.

TREATMENT
Fortunately, this patient’s hypopigmentation will resolve quickly with treatment of her eczema, using a low-strength steroid cream (eg, hydrocortisone 2.5% cream or ointment). But a good portion of the “treatment” of AD is done by educating the family about the nature of the condition, as well as providing reassurance about the absence of the more serious items in the differential.

Author and Disclosure Information

 

Joe R. Monroe, MPAS, PA-C

Issue
Clinician Reviews - 22(10)
Publications
Topics
Page Number
5
Legacy Keywords
dermatology, leprosy, vitiligo, lesion, ringworm, steriods, infant, clotrimazole, atopy, eczema, hypopigmentation, postinflammatory, pigment loss, child, central hypopigmentation,
Sections
Author and Disclosure Information

 

Joe R. Monroe, MPAS, PA-C

Author and Disclosure Information

 

Joe R. Monroe, MPAS, PA-C

ANSWER
The correct answer is postinflammatory hypopigmentation (choice “c”), in this case secondary to eczema in a classic antecubital location. Leprosy (choice “a”) is more common than one might imagine, but it does not appear overnight and does not involve overt inflammation. Vitiligo (choice “b”) does not appear suddenly and rarely involves the type of inflammation seen in this case. Lichen sclerosis et atrophicus (choice “d”) is an inflammatory condition that presents with hypopigmentation and epidermal atrophy; however, it is gradual in onset and would not exhibit papulosquamous inflammation.

DISCUSSION
The more color in the skin, the more the loss of that color stands out. Patients and families with darker skin are often understandably upset by the contrast. Providers need a differential for pigment loss, including the items mentioned—some of which have the potential to be dreadfully serious.

Two relevant facts stand out in this case: the rapidity of onset and the history of eczema, in which secondary pigment loss can occur. As mentioned, it is especially obvious in those with darker skin. Fortunately, once the eczema calms down, the hypopigmentation resolves and normal color returns.

Paradoxically, it’s not at all unusual to see postinflammatory hyperpigmentation, especially in those with skin of types IV and V (eg, African-Americans, some Hispanics, and those of Indian ancestry). Eczema is a common cause, but the inflammation can be from almost any source, including trauma, burns, or even acne.

Had this patient’s diagnosis not been obvious, a biopsy might have been indicated due to the serious nature of some of the items in the differential. Vitiligo, for example, can be very disfiguring, especially on a dark-skinned individual. It tends to become widespread and permanent, unless it’s caught and treated early on. Other conditions involving hypopigmentation include sarcoidosis, lupus, and morphea.

All of these conditions are unusual, if not rare, compared with atopic dermatitis (AD), which this patient has. AD is so common that almost 20% of newborns develop it. Eczema is one of the more typical manifestations, along with dry, sensitive skin, seasonal allergies, and reactive airway disease. Corroboration of the diagnosis is usually easily accomplished by taking a family history.

TREATMENT
Fortunately, this patient’s hypopigmentation will resolve quickly with treatment of her eczema, using a low-strength steroid cream (eg, hydrocortisone 2.5% cream or ointment). But a good portion of the “treatment” of AD is done by educating the family about the nature of the condition, as well as providing reassurance about the absence of the more serious items in the differential.

ANSWER
The correct answer is postinflammatory hypopigmentation (choice “c”), in this case secondary to eczema in a classic antecubital location. Leprosy (choice “a”) is more common than one might imagine, but it does not appear overnight and does not involve overt inflammation. Vitiligo (choice “b”) does not appear suddenly and rarely involves the type of inflammation seen in this case. Lichen sclerosis et atrophicus (choice “d”) is an inflammatory condition that presents with hypopigmentation and epidermal atrophy; however, it is gradual in onset and would not exhibit papulosquamous inflammation.

DISCUSSION
The more color in the skin, the more the loss of that color stands out. Patients and families with darker skin are often understandably upset by the contrast. Providers need a differential for pigment loss, including the items mentioned—some of which have the potential to be dreadfully serious.

Two relevant facts stand out in this case: the rapidity of onset and the history of eczema, in which secondary pigment loss can occur. As mentioned, it is especially obvious in those with darker skin. Fortunately, once the eczema calms down, the hypopigmentation resolves and normal color returns.

Paradoxically, it’s not at all unusual to see postinflammatory hyperpigmentation, especially in those with skin of types IV and V (eg, African-Americans, some Hispanics, and those of Indian ancestry). Eczema is a common cause, but the inflammation can be from almost any source, including trauma, burns, or even acne.

Had this patient’s diagnosis not been obvious, a biopsy might have been indicated due to the serious nature of some of the items in the differential. Vitiligo, for example, can be very disfiguring, especially on a dark-skinned individual. It tends to become widespread and permanent, unless it’s caught and treated early on. Other conditions involving hypopigmentation include sarcoidosis, lupus, and morphea.

All of these conditions are unusual, if not rare, compared with atopic dermatitis (AD), which this patient has. AD is so common that almost 20% of newborns develop it. Eczema is one of the more typical manifestations, along with dry, sensitive skin, seasonal allergies, and reactive airway disease. Corroboration of the diagnosis is usually easily accomplished by taking a family history.

TREATMENT
Fortunately, this patient’s hypopigmentation will resolve quickly with treatment of her eczema, using a low-strength steroid cream (eg, hydrocortisone 2.5% cream or ointment). But a good portion of the “treatment” of AD is done by educating the family about the nature of the condition, as well as providing reassurance about the absence of the more serious items in the differential.

Issue
Clinician Reviews - 22(10)
Issue
Clinician Reviews - 22(10)
Page Number
5
Page Number
5
Publications
Publications
Topics
Article Type
Display Headline
Is Leprosy the Cause of This Girl's Lesion?
Display Headline
Is Leprosy the Cause of This Girl's Lesion?
Legacy Keywords
dermatology, leprosy, vitiligo, lesion, ringworm, steriods, infant, clotrimazole, atopy, eczema, hypopigmentation, postinflammatory, pigment loss, child, central hypopigmentation,
Legacy Keywords
dermatology, leprosy, vitiligo, lesion, ringworm, steriods, infant, clotrimazole, atopy, eczema, hypopigmentation, postinflammatory, pigment loss, child, central hypopigmentation,
Sections
Questionnaire Body

 

 

The parents of this 8-month-old infant are alarmed by skin changes that occurred practically overnight on the child’s arm—especially since the child’s grandparents suggested it might represent vitiligo or even leprosy. The child’s pediatrician thought “ringworm” was more likely, but the clotrimazole cream he recommended was no help. The child has an extensive history of atopy, including eczema affecting the trunk and face. The parents have used topical steroid cream on the affected areas with some good effect, but the loss of color in the antecubital site has made them reluctant to use the product on this new site. Examination shows a papulosquamous lesion, 3.5 cm in diameter, on the left lateral antecubital area, with marked central hypopigmentation. The child and her parents are Vietnamese, with type IV skin, making the pigment loss all the more obvious. The periphery of the lesion, in addition to being bumpy and scaly, is moderately inflamed. The rest of the child’s skin is dry but otherwise unremarkable.

 

Disallow All Ads